Jump to content

xuanhoan23112002's Content

There have been 95 items by xuanhoan23112002 (Search limited from 08-06-2020)



Sort by                Order  

#707185 Tìm giá trị nhỏ nhất của: $S=a_{1}^{2}+a_{2...

Posted by xuanhoan23112002 on 28-04-2018 - 15:19 in Bất đẳng thức - Cực trị

Problem: Cho $a_{1}, a_{2},...,a_{19}$ là các số tự nhiên thỏa mãn: $a_{1}+a_{2}+...+a_{19}=26.$ Tìm giá trị nhỏ nhất của: $S=a_{1}^{2}+a_{2}^{2}+...+a_{19}^{2}.$

 




#709807 Bất đẳng thức

Posted by xuanhoan23112002 on 03-06-2018 - 07:54 in Bất đẳng thức - Cực trị

Cho a, b, c là các số thực không âm. Chứng minh rằng:

$3(a+b+c)\geq 2(\sqrt{a^2+bc}+\sqrt{b^2+ca}+\sqrt{c^2+ab})$




#711677 $f(x)=ax^4+bx+c> 0 \forall x> 0$

Posted by xuanhoan23112002 on 27-06-2018 - 16:55 in Đa thức

Cho $a\neq 0$ và $f(x)=ax^4+bx+c> 0 \forall x> 0$

CMR: $f(x)$ được biểu diễn ở dạng tổng bình phương của 2 tam thức bậc hai.




#709553 số học

Posted by xuanhoan23112002 on 30-05-2018 - 07:32 in Số học

Từ giả thiết ta thấy ngay a, b, c đều là các số lẻ mà một số chính phương lẻ chia 8 dư 1

Từ nhận xét trên: $a^{30}+b^{4}+c^{2018}\equiv 3$ (mod 8)




#707928 Bất đẳng thức

Posted by xuanhoan23112002 on 08-05-2018 - 21:35 in Bất đẳng thức và cực trị

BĐT$\Leftrightarrow \frac{a^2}{ab+ac}+\frac{b^2}{bc+bd}+\frac{c^2}{cd+ca}+\frac{d^2}{da+db}\geq 2$

Ta có VT$\geq \frac{(a+b+c+d)^2}{(a+d)(b+c)+(c+d)(a+b)}$( theo BĐT Cauchy-Schwarz)

Mà cũng theo BĐT AM-GM ta cũng có $(a+d)(b+c)+(a+b)(c+d)\leq \frac{(a+b+c+d)^2}{2}$

Do đó VT$\geq 2$

Vậy bất đẳng thức được chứng minh. Đẳng thức xảy ra$\Leftrightarrow$ $a= b= c= d> 0$




#709566 Số chính phương

Posted by xuanhoan23112002 on 30-05-2018 - 09:39 in Số học

Bài toán này sử dụng phương pháp bước nhảy Viete. Các bài viết khác về bước nhảy Viete trên VMF

http://diendantoanho...ước-nhảy-viete/

Lời giải của bài toán trên bạn có thể tham khảo ở đây: http://math.stackexc...-its-an-integer




#705630 Số học

Posted by xuanhoan23112002 on 12-04-2018 - 20:09 in Số học

Tìm các số nguyên dương n sao cho: Với mọi a, b là các số nguyên dương, nếu a2b+1 chia hết cho n thì a2+b cũng chia hết cho n




#710109 Đề thi tuyển sinh vào lớp 10 THPT tỉnh Bắc Giang năm học 2018-2019

Posted by xuanhoan23112002 on 06-06-2018 - 15:07 in Tài liệu - Đề thi

Câu 5: Ta có:

$P=\frac{81x^2+18225x+1}{9x}-\frac{6\sqrt{x}+8}{x+1}\geq \frac{18x}{9x}-\frac{9x+9}{x+1}+2025= 2018$ (bất đẳng thức Cauchy)

Đẳng thức xảy ra $\Leftrightarrow x=\frac{1}{9}> 0$

Vậy $MinP=2018\Leftrightarrow x=\frac{1}{9}$




#705136 Chứng minh rằng với mọi số thực dương $a,b>2$ thì $2^a-1...

Posted by xuanhoan23112002 on 06-04-2018 - 22:59 in Số học

Bài toán sai khi a chia hết cho b 

Nếu a không chia hết cho b. Đặt a=bq+r(0<r<b)

Sử dụng phản chứng để suy ra 2r - 1 chia hết cho 2b - 1 (điều này vô lí do 0<2r - 1<2b - 1)

Từ đó ta có điều phải chứng minh.




#709479 Bài tập về đa thức

Posted by xuanhoan23112002 on 29-05-2018 - 08:53 in Đa thức

Bài 1: Cho đa thức $f(x)=x^{2018}+\sum a_ix^{i}($a_i\in {-1,1}, $\forall i\in \left \{ 0,1,...,2017 \right \}$$)$ không có nghiệm thực. Tìm số lớn nhất các hệ số = -1 trong f(x)

Bài 2: Tìm đa thức P(x) hệ số thực thỏa mãn:

$(P(x))^{3}-3(P(x))^{2}=P(x^{3})-3P(-x)$, với mọi x là số thực




#710198 AP vuông góc với IJ

Posted by xuanhoan23112002 on 07-06-2018 - 14:17 in Hình học

Cho tam giác ABC với AC > AB. Các đường cao BB, CCcủa tam giác cắt nhau tại H. Gọi M, N lần lượt là trung điểm của BC', CB'. MH cắt đường tròn ngoại tiếp tam giác CHB' tại I; N

H cắt đường tròn ngoại tiếp tam giác BHC' tại J. Giả sử P là trung điểm cạnh BC. Chứng minh: AP vuông góc với IJ




#710187 Tâm đường tròn ngoại tiếp tứ giác ABCD nằm trên SI

Posted by xuanhoan23112002 on 07-06-2018 - 11:51 in Hình học

Cho tứ giác ABCD ngoại tiếp đường tròn tâm I. Giả sử bên trong tứ giác ta vẽ được 4 đường tròn bằng nhau và cùng đi qua 1 điểm S, và mỗi đường tròn tiếp xúc với 2 cạnh liên tiếp của tứ giác đó. Chứng minh tứ giác ABCD nội tiếp 1 đường tròn và tâm đường tròn đó nằm trên SI.




#712421 $2^n-1$ là số nguyên tố

Posted by xuanhoan23112002 on 12-07-2018 - 22:34 in Số học

Bài này dùng phản chứng thôi.




#709360 Cho $x, y, z > 0$ và $x+y+z= 1$ . Chứng minh

Posted by xuanhoan23112002 on 27-05-2018 - 15:33 in Bất đẳng thức và cực trị

Theo bất đẳng thức Schur ta có:

$(x+y+z)^3+9xyz\geq 4(x+y+z)(xy+yz+zx)$

$\Leftrightarrow 9xyz\geq 4(xy+yz+zx)-1$

$\Leftrightarrow 5xyz+1\geq 4(xy+yz+zx-xyz)$

Theo bất đẳng thức AM-GM ta có:

$xyz\leq \frac{(x+y+z)^3}{27}= \frac{1}{27}$

$\Rightarrow xy+yz+zx-xyz\leq \frac{8}{27}$

Vậy bất đẳng thức được chứng minh.




#705203 Trong một giải đấu bóng đá có 10 đội tham gia theo thể thức mỗi đội đều gặp đ...

Posted by xuanhoan23112002 on 08-04-2018 - 08:27 in Tổ hợp và rời rạc

Ta chứng minh bài toán bằng phản chứng( cả 1 và 2 đều không xảy ra)

Gọi 10 đội bóng là a(i là số tự nhiên và i chạy từ 1 đến 10)

Giả sử a10 là đội bóng có số trận thua nhiều nhất

Khi đó nếu tồn tại giá trị i từ 1 đến 9 mà a10 thang ai thì tất cả cả đội bóng mà a10 thua thì ai cũng thua (vô lí do a10 có số trận thua nhiều nhất)

Suy ra a10 thi đấu với các đội còn lại chỉ có thể hòa hoặc thua

Mà theo gia sư điều kiện 2 không xảy ra nên a10 thua ít nhất 7 đội là aj (j chạy từ 1 đến 7)

Lập luận tương tự như trên với a7 là đội có số trận thua nhiều nhất trong 7 đội trên thì a7 phải thừa ít nhất 4 đội giả sử là: a1, a2, a3, a4.

Lập luận tương tự như trên với a4 là đội có số trận thua nhiều nhất trong 4 đội trên thì a4 phải thừa ít nhất 1 đội giả sử là: a1

Như vậy ta tìm được 4 đội: a1, a4, a7, a10, lập thành 4 đội thỏa mãn điều kiện 1( mâu thuẫn với giả sử)

Do đó giả sử sai. Ta có điều phải chứng minh




#705205 Tìm a? đề hàm số có đạo hàm tại x=-1

Posted by xuanhoan23112002 on 08-04-2018 - 08:32 in Hàm số - Đạo hàm

Ta phải xét tính liên tục của hàm số và đặt điều kiện đạo hàm trái = đạo hàm phải bạn nhé




#704920 Lập phương trình đt d qua M

Posted by xuanhoan23112002 on 04-04-2018 - 22:14 in Phương pháp tọa độ trong mặt phẳng

Cho điểm M(2;1). Lập d đi qua M cắt Ox, Oy tại A, B sao cho khoảng cách từ O đến d là max

d:ax+by-2a-b=0

(a,b khác 0)

Từ đây ta tìm được tọa độ giao điểm của (d) với Ox, Oy theo a, b

Áp dụng hệ thức lượng trong tam giác vuông tính được khoảng cách từ O đến d theo a,b

Tìm max của giá trị.




#705140 chứng minh

Posted by xuanhoan23112002 on 06-04-2018 - 23:36 in Số học

Lời giải nhanh nhất cho bài toán này là cách áp dụng định lý Fermat nhỏ:

x- x chia hết cho 5

Dễ dàng chứng minh x5 - x chia hết cho 2

Như vậy ta cũng có điều phải chứng minh




#704997 tìm giá trị của x để a nguyên

Posted by xuanhoan23112002 on 05-04-2018 - 21:04 in Đại số

PP làm: lấy tử chia mẫu được phần nguyên và phân số rồi sử dụng tính chia hết




#705137 chứng minh

Posted by xuanhoan23112002 on 06-04-2018 - 23:24 in Số học

chứng minh rằng với mọi n$\epsilon$$\mathbb{Z}$ thì $n^{5}$ và n có chữ số tận cùng giống nhau 

n5 - n =n(n-1)(n+1)(n2+1)

Dễ dàng chứng minh n5-n chia hết cho 2

Với n chia hết cho 5, chia 5 dư 1 hoặc 4 thì n5-n chia hết cho 5

Với n chia 5 dư 2 hoặc 3 thì n2+1 chia hết cho 5

Như vậy ta có n5-n chia hết cho 5, n5-n chia hết cho 2 và gcd(5,2)=1

Nên n5-n chia hết cho 10

Hay n5 và n có chữ số tận cùng giống nhau




#709289 Bất đẳng thức trong đề thi vào lớp 10 Nam Định năm 2018

Posted by xuanhoan23112002 on 26-05-2018 - 15:38 in Bất đẳng thức và cực trị

Cho a, b, c là các số thực dương thỏa mãn: $a^2+b^2+c^2+abc=4$. Chứng minh rằng:

$2a+b+c\leq \frac{9}{2}$




#709300 Bất đẳng thức trong đề thi vào lớp 10 Nam Định năm 2018

Posted by xuanhoan23112002 on 26-05-2018 - 17:55 in Bất đẳng thức và cực trị

Đây là đề của ban xã hội bạn ạ.

Còn đây là lời giải của mình cho bài toán này các bạn có thể tham khảo:

Coi phương trình trên là phương trình bậc 2 ẩn a theo công thức nghiệm ta được

$a=\frac{-bc+\sqrt{(4-b^2)(4-c^2)}}{2}$

Áp dụng bất đẳng thức Cauchy cho căn thức trong biểu thức trên, ta có:

$a\leq \frac{-bc+\frac{4-b^2+4-c^2}{2}}{2}= \frac{8-(b+c)^2}{4}$

Từ đó ta có: $2a+b+c=\frac{8-(b+c)^2+2(b+c)}{2}=\frac{9-(b+c-1)^2}{2}\leq \frac{9}{2}$

P/s: Mình nghĩ đây là cách ngắn nhất và có thể thay số 2 trong đề bài bởi các số khác vẫn có thể giải tương tự.




#709756 Tìm GTLN của $2x^2-3xy-2y^2$

Posted by xuanhoan23112002 on 02-06-2018 - 07:23 in Bất đẳng thức và cực trị

Đặt $A=2x^2-3xy-2y^2$

$\Leftrightarrow$$A-3=2x^2-3xy-2y^2-\frac{1}{12}(25x^2-20xy+40y^2)$

$\Leftrightarrow$$A-3=-\frac{1}{12}x^2-\frac{4}{3}xy-\frac{16}{3}y^2$

$\Leftrightarrow$$A-3=-\frac{1}{12}(x+8y)^2\leq 0$

$\Leftrightarrow A\leq 3$

Đẳng thức xảy ra $\Leftrightarrow (x, y)=(\frac{4\sqrt{2}}{5}, -\frac{\sqrt{2}}{10})$ hoặc $(x, y)=(-\frac{4\sqrt{2}}{5}, \frac{\sqrt{2}}{10})$

Vậy Max của $2x^2-3xy-2y^2=3$




#709806 Tìm GTLN của $2x^2-3xy-2y^2$

Posted by xuanhoan23112002 on 03-06-2018 - 07:30 in Bất đẳng thức và cực trị

Ta có:

$A-36a=(2-25a)x^2-(3+20a)xy-(2-40a)y^2$

Coi phương trình trên là phương trình bậc 2 ẩn x tìm giá trị của a sao cho phương trình có nghiệm kép tức là$\Delta =0$ (Chú ý: Tìm giá trị lớn nhất thì $A-36a$ mang dấu trừ của 1 bình phương đủ nên $2-25a<0, 2-40a<0$)

Từ đó tìm được: $a=\frac{1}{12 }$




#709371 CMR: $\sum \frac{ab+c^{2}}{a+b}...

Posted by xuanhoan23112002 on 27-05-2018 - 17:05 in Bất đẳng thức và cực trị

Ta có: $\sum \frac{ab+c^2}{a+b}+\sum c= \sum \frac{(c+a)(c+b)}{a+b}\geq 2(a+b+c)$ (bất đẳng thức AM-GM)

$\Rightarrow \sum \frac{ab+c^2}{a+b}\geq a+b+c$

Đẳng thức xảy ra $\Leftrightarrow a=b=c> 0$

Vậy bất đẳng thức được chứng minh.